Domanda:
Come posso rappresentare R quadrato in forma di matrice?
Luna
2012-07-27 20:15:28 UTC
view on stackexchange narkive permalink

Questa domanda è il seguito di una domanda precedente.


Fondamentalmente, volevo studiare in quali condizioni regrediamo a $ x_1 $ , otterremo $ \ small R ^ 2 $ del 20%.

Come primo passo per affrontare questo problema, la mia domanda è: come esprimo $ \ small R ^ 2 $ in forma di matrice?

Quindi cercherò di esprimere "$ \ small R ^ 2 $ di regressione dei residui a $ x_1 $ "utilizzando il modulo di matrice.

Inoltre, come posso aggiungere i pesi di regressione nell'espressione?

Re: * Fondamentalmente volevo studiare in quali condizioni quando regrediamo i residui a x1, otterremo un R-quadrato del 20% *, se la regressione è una normale regressione dei minimi quadrati e $ x_1 $ è stato incluso nel modello , quindi la risposta non è mai, e ti ho mostrato nella mia risposta [qui] (http://stats.stackexchange.com/questions/32471/how-can-you-handle-unstable-beta-estimates-in-linear- regression-with-high-mul). Se stai effettivamente chiedendo qualcos'altro, ti preghiamo di chiarire.
Ciao Macro, perché ho pesi nella regressione. Volevo essere in grado di derivare qualcosa dallo studio dello studio R ^ 2. Questo è il motivo per cui si richiede l'espressione in forma matrice. Grazie!
Ciao @Luna - OK, ma in questo post non si fa menzione dei pesi. Che tipo di pesi? Puoi modificare il post per chiarire?
Avevo menzionato i pesi in quel thread ... qui sto cercando un modulo generico - una volta appreso il modulo generico, posso aggiungere i pesi da solo ... giusto?
https://stats.stackexchange.com/q/351200/119261
Due risposte:
Charlie
2012-07-28 00:36:37 UTC
view on stackexchange narkive permalink

Abbiamo $$ \ begin {align *} R ^ 2 = 1 - \ frac {\ sum {e_i ^ 2}} {\ sum {(y_i - \ bar {y}) ^ 2}} = 1 - \ frac {e ^ \ prime e} {\ tilde {y} ^ \ prime \ tilde {y}}, \ end {align *} $$ dove $ \ tilde {y} $ è un vettore $ y $ sminuito.

Ricorda che $ \ hat {\ beta} = (X ^ \ prime X) ^ {- 1} X ^ \ prime y $, implicando che $ e = y - X \ hat {\ beta} = y - X (X ^ \ prime X) ^ {- 1} X ^ \ prime y $. La regressione su un vettore di 1s, scritto come $ l $, fornisce la media di $ y $ poiché il valore previsto e i residui di quel modello producono valori $ y $ sminuiti; $ \ tilde {y} = y - \ bar {y} = y - l (l ^ \ prime l) ^ {- 1} l ^ \ prime y $.

Sia $ H = X ( X ^ \ prime X) ^ {- 1} X ^ \ prime $ e sia $ M = l (l ^ \ prime l) ^ {- 1} l ^ \ prime $, dove $ l $ è un vettore di 1. Inoltre, sia $ I $ una matrice di identità della dimensione richiesta. Quindi abbiamo

$$ \ begin {align *} R ^ 2 & = 1- \ frac {e ^ \ prime e} {\ tilde {y} ^ \ prime \ tilde {y}} \ \ & = 1 - \ frac {y ^ \ prime (I - H) ^ \ prime (IH) y} {y ^ \ prime (I - M) ^ \ prime (IM) y} \\ & = 1 - \ frac {y ^ \ prime (IH) y} {y ^ \ prime (IM) y}, \ end {align *} $$

dove la seconda riga deriva dal fatto che $ H $ e $ M $ (e $ I $) sono idempotenti.

Nel caso ponderato, sia $ \ Omega $ la matrice di ponderazione utilizzata nella funzione obiettivo OLS, $ e ^ \ prime \ Omega e $. Inoltre, sia $ H_w = X \ Omega ^ {1/2} (X ^ \ prime \ Omega X) ^ {- 1} \ Omega ^ {1/2} X ^ \ prime $ e $ M_w = l \ Omega ^ {1/2} (l ^ \ prime \ Omega l) ^ {- 1} \ Omega ^ {1/2} l ^ \ prime $. Quindi, $$ \ begin {align *} R ^ 2 & = 1 - \ frac {y ^ \ prime \ Omega ^ {1/2} (I-H_w) \ Omega ^ {1/2} y} {y ^ \ prime \ Omega ^ {1/2} (I-M_w) \ Omega ^ {1/2} y}, \ end {align *} $$

+1 È bello (ed elegante) vedere la varianza al denominatore emergere dalla regressione rispetto a una costante.
molte grazie! Ho votato per te. Inoltre, come aggiungere pesi di regressione all'espressione? Grazie!
@Luna, se questo post risponde alla tua domanda e fornisce le informazioni di cui hai bisogno, dovresti considerare di * accettarlo * (facendo clic sul segno di spunta alla sua sinistra) e votarlo.
@Charlie, Penso che tu possa aver accidentalmente eliminato $ y $ dalla tua prima equazione per $ \ hat \ beta $. Inoltre, non seguo esattamente la tua equazione per $ R ^ 2 $, sono abituato a vedere $ \ sum (\ hat y_i- \ bar y) ^ 2 / \ sum (y_i - \ bar y) ^ 2 $ oppure $ 1 - (\ sum (y_i- \ hat y_i) ^ 2 / \ sum (y_i- \ bar y) ^ 2) $. Sto interpretando il tuo $ e ^ 2 $ come $ \ sum (y_i- \ hat y_i) ^ 2 $, quindi sono confuso; c'è un modo che può renderlo più chiaro per me?
@gung, Hai ragione, avevo $ R ^ 2 $ definito in modo errato. Spero che sia corretto ora. Grazie!
+1, grazie, @Charlie, ora ha molto più senso per me.
Ben
2019-05-07 05:32:48 UTC
view on stackexchange narkive permalink

Puoi scrivere il coefficiente di determinazione come una semplice forma quadratica dei valori di correlazione tra le singole variabili (vedi questa risposta per i dettagli). Considera un regressione lineare multipla con $ m $ vettori esplicativi e un termine di intercetta. Lascia che $ r_i = \ mathbb {Corr} (\ mathbf {y}, \ mathbf {x} _i) $ e $ r_ {i, j} = \ mathbb {Corr} (\ mathbf {x} _i, \ mathbf {x} _j) $ e definisci:

$$ \ boldsymbol {r} _ {\ mathbf {y}, \ mathbf {x}} = \ begin {bmatrix} r_1 \\ r_2 \\ \ vdots \\ r_m \ end {bmatrix} \ quad \ quad \ quad \ boldsymbol {r} _ {\ mathbf {x}, \ mathbf {x}} = \ begin {bmatrix} r_ {1,1} & r_ {1,2} & \ cdots & r_ {1, m} \\ r_ {2,1} & r_ {2,2} & \ cdots & r_ {2, m} \\ \ vdots & \ vdots & \ ddots & \ vdots \\ r_ {m, 1} & r_ {m, 2} & \ cdots & r_ {m, m} \\ \ end {bmatrix}. $$

Con un po 'di algebra lineare si può dimostrare che:

$$ R ^ 2 = \ boldsymbol {r} _ {\ mathbf {y}, \ mathbf {x}} ^ \ text {T} \ boldsymbol {r} _ {\ mathbf {x}, \ mathbf {x}} ^ {- 1} \ boldsymbol {r} _ {\ mathbf {y}, \ mathbf {x}}. $$

La radice quadrata del coefficiente di determinazione fornisce il coefficiente di correlazione multipla, che è un'estensione multivariata della correlazione assoluta.



Questa domanda e risposta è stata tradotta automaticamente dalla lingua inglese. Il contenuto originale è disponibile su stackexchange, che ringraziamo per la licenza cc by-sa 3.0 con cui è distribuito.
Loading...